Pls help I will give 20 points if anwser is correct

-8 3/4 divide 2 1/6

Answers

Answer 1

Answer: Improper form: -106/26 Mixed fraction form: -4 1/26

Decimal form: -4.03846153...

Step-by-step explanation: Simplify the expression, if possible, by cancelling the common factors.


Related Questions

If 2 angles in a triangle are 71 degrees and 56 degree, what is the measure of the third angle?

Answers

Answer:

53

Step-by-step explanation:

Sol'n

Let the remaining angle of a triangle be x.

x+71+56=180 (Sum of angles of a triangle is 180°)

or, x+127=180

or, x=180-127

.:, x=53°

.:, The measure of third angle is 53°

Pamela is 7 years younger than Jiri. The sum of their ages is 63. What is Jiri's age?

Answers

Answer: 35

Step-by-step explanation: (lol I read the question wrong). Basically subtract 7 from 63, then divide 56 by 2 to find Pamela’s age. Then add 28+7 because Jiri is 7 years older than Pamela

Pamela is 28 and Jiri is 35. This would make Pamela 7 years younger than Jiri, as well as ensuring the sum of their ages is 63 since 28+35=63.

Y- -7= -1(x-1)
What is this written in slope intercept form

Answers

Answer:

Y= - x - 6

Step-by-step explanation:

Cal has $400 in his saving account and deposits $25 per month. Marla has $100 in her savings account and deposits $50 per month. After how many months will they have the same amount in their savings account?

Answers

Answer:

11 months

Step-by-step explanation:

If you want to know the probability of spinning a spinner in a game that has 8 equal-sized
wedges and it lands on red, which represents one of the those wedges. You determine that probability to be 1/8 or 0.125.

Identify if the following is experimental, classical, or subjective simple probability:

Answers

the answer is 1/8. Because you divide how many pieces there are by that same number.

I think it’s 1/8 sorry if I’m wrong

13. A manager is comparing the drive-through service times for two fast-food restaurants, and the following results (in seconds) were obtained. Find the range, variance, and standard deviation for each of the two samples, then
compare the two sets of results
Restaurant A 120 123 153 128 124 118 154 110

Restaurant B 115 126 147 156 118 110 145 137
Restaurant A: 44 sec; 298.34 sec?; 17.27 sec
Restaurant B: 46 sec; 285.64 sec2; 16.90 sec
There is more variation in the times for restaurant A.
Restaurant A: 44 sec; 260.79 sec?; 16.15 sec
Restaurant B: 46 sec; 285.64 sec2; 16.90 sec
There is more variation in the times for restaurant B.
Restaurant A: 44 sec; 260.79 sec2; 16.15 sec
Restaurant B: 46 sec; 245.64 sec?; 15.67 sec
There is more variation in the times for restaurant A.
Restaurant A: 46 sec; 260.79 sec2; 16.15 sec
Restaurant B: 44 sec; 285.64 sec2; 16.90 sec
There is more variation in the times for restaurant B.

Answers

Answer:

The answer is A.

Step-by-step explanation:

Which list is in order from least to greatest? A. 29, 15−−√, π, 4.1 B. 29, π, 15−−√, 4.1 C. 4.1, 15−−√, π, 29 D. 4.1, π, 15−−√, 29

Answers

Answer:

From least to greatest;

[tex]\pi \:, \sqrt{15} , \: 4.1 \: ,29[/tex]

Step-by-step explanation:

[tex]\pi = 3.14 \\ \sqrt{15} = 3.87 \\ 4.1 = 4.1 \\ 29 = 29[/tex]

I hope I helped you^_^

10% of 30 (this is for a quiz, thanks if u can help)

Answers

The answer would be 3, to find this you would do .10 x 30 to get 3

Answer:

3

Step-by-step explanation:

3 * 10 = 30

Hope I could help!

simplify this please!!​

Answers

Answer:

[tex] {5}^{ \frac{1}{2} } \times {5}^{ \frac{1}{3} } = {5}^{ \frac{5}{6} } [/tex]

the circumcenter of a triangle is equidistant from the

Answers

Answer:

The circumcenter of a triangle is a point that is equidistant from all three vertices. The circumscribed circle is a circle whose center is the circumcenter and whose circumference passes through all three vertices.

Step-by-step explanation:

We can conclude that the circumcenter of a triangle is equidistant from the three vertices of the triangle.

What is the Circumcenter of a Triangle?

The circumcenter of a triangle can be said to be a point where all three perpendicular bisectors of the sides of the triangle intersect. At this point, all three vertices of the triangle are equidistant.

This means that, each vertices are far from the circumcenter at equal distance.

Therefore, we can conclude that the circumcenter of a triangle is equidistant from the three vertices of the triangle.

Learn more about the circumcenter on:

https://brainly.com/question/21299234

A 1 pound bag of sunflower seeds provides 96 grams of protein

Answers

Well what’s the question lol-

How many liters of a 50% acid solution must be mixed with a 30% acid solution to get 180 L of a 40% acid solution?

Answers

Answer:

ninety liters

Step-by-step explanation:

Ninety liters of the 50% acid solution have to be mixed with 90 liters of the 30% solution to yield 180 liters of 40% solution.

find three consecutive even integers whose sum 72

Answers

22, 24, 26

Step-by-step explanation:

2n - 2 + 2n + 2n + 2 = 72

6n = 72

n = 72/6

n = 12

∴ so three consecutive even integers whose sum is 72 :

2n - 2

= 2(12) - 2

= 24 - 2

= 22

2n

= 2(12)

= 24

2n + 2

= 2(12) + 2

= 24 + 2

= 26

verification

22 + 24 + 26

= 46 + 26

= 72 ✓✓

What is this expression in simplest form?
x²+x-2/x³-x²+2x-2

Answers

Answer:

Simplified to lowest possible would be [tex]\frac{x +2}{x^{2} + 2}[/tex]

Step-by-step explanation:

What is 39.2÷2? equal to?

Answers

Answer:

19.6 is the correct answer

Answer:

19.6

Step-by-step explanation:

39.2/2

19.6

what the slope of a line

Answers

Answer:

The slope of a line is a number that measures its "steepness", usually denoted by the letter m. It is the change in y for a unit change in x along the line.

Hope it helps.

find the value of a + 1a​

Answers

Answer:

2a

Step-by-step explanation:

a+1a is the same as a+a=2a

2. On the graph below you will find two different triangles, and three points identified along the line.
(a) Identify the coordinates of Point A, B, & C? Explain.
(b) Does each triangle have the same change in x-values? Explain.
(c) Does each triangle have the same change in y-values? Explain.
(d) Does each triangle have the same ratio of change in y-values over change in x-values? Explain.

Answers

#a

Co-ordinates are

A(0,0)B(3,3)C(9,6)

#b

No,Both triangles are congruent not same.

#c

No! Both triangles are not same

#d

Yes ratio is same as they are congruent

Answer:

Question A:

A.) (0,0)

B.) (3,3)

C.) (9,6)

Question B:

No, both triangles are congruent, not the same change in x-values.

Question C:

No, both triangles do not have are not the same change in y-values.

Question D:

Yes, their ratios of change are the same.

Find the slope of the line.

Answers

Answer:

The slope is y = x

Step-by-step explanation:

You can find the slope by using the rise over run method with two points.Let's take (1,1) and (2,2). From the first point to the second point, (1,1) goes up one unit and to the right one unit.So the rise over run is 1/1, or 1, which means the slope is 1.

Answer:

m = 1x

Step-by-step explanation:

This is the easiest slope you'll ever have to calculate - if you even need to as it's self explanatory looking at it! Slope = rise/run, thus slope = 1/1 = 1.

NEEDS TO BE CORRECT PLZ HELP

Answers

C

Step-by-step explanation:

Linear pair

it's hope it helpful to you

Answer:

c

Step-by-step explanation:

its a pair

is 9:2 equivalent to 18:4

Answers

Answer:

That is true

Step-by-step explanation:

9:2 = 18:4

9x2=18

2x2=4

Thus, this answer is true.

Answer:

Yes, because if you simplify 18:4, it would simplify to 9/2. Both ratios are equivalent.

Step-by-step explanation:

What is the answer tho this problem 315+1.4x

Answers

Answer:

316.4     in the calculator it came out like this

Step-by-step explanation:

Solve the equation –x^2 -6x=-2x^2 to the nearest tenth.

Answers

Answer:

The answer is x = 3

Step-by-step explanation:

edge2020

Write an equation for the line parallel to the given line that contains C.
C(-1,5); y = 8/9x+2
*

Answers

Answer:

y = 8/9x + 53/9

Step-by-step explanation:

5 = 8/9(-1) + b

45/9 = -8/9 + b

b = 53/9

WORTH 50 POINTS HELP ASAP WILL GIVE BRAINLIEST!
The price of a new pair of shoes is $52.00. If you order them directly from the company on the internet, you can save money and the price is only $42.64. What is the percent change?
18% decrease
22% decrease
18% increase
22% increase

Answers

18% decrease

Hope this helped you-have a good day bro cya)

Answer:

The percent change is an 18% decrease.

Step-by-step explanation:

how to multiply mixed fractions with whole numbers

Answers

Answer:

Step-by-step explanation:

Example: (1 2/3) (2 3/5)

Turn to improper fractions

1 2/3= 5/3   2 3/5= 13/5

5/3 x 13/5=?

5x13= 65

3x5= 15

Simplify/divide: 65/15 = 4 1/3

Another example. (2 5/6)(3)

Improper fraction:  2 5/6= 17/6

Turn Whole number to fraction: 3= 3/1

17/6 x 3/1

17x3= 51

6x1= 6

51/6

Simplify/divide: 8 1/2

Evaluate the function.

Answers

-87 is the correct answer

Triangle ABC is transformed to create triangle MNO and triangle PQR as described below.

MNO represents the translation 2 units left and 2 units down of triangle ABC.
PQR represents the dilation of triangle ABC by a scale factor of 2.
Which of these is correct?

Answers

The missing options are located here; https://brainly.com/question/18156103

The option that correctly describes the transformation that took place with triangle ABC is; Option A.

In transformations, translation simply means shifting an object from one position to the other while dilation simply means increasing or decreasing an object by a scale factor.This means that translation preserves the lengths and angles of the object while the lengths and angles change with dilation.

Option A; Triangle MNO represents a translated triangle and as such the lengths and angles remain unchanged and so this option is correct.

Option B; Not true because both lengths and sides are preserved as stated above.

Option C; Transformation to PQR denotes dilation by a factor of 2 and as such the lengths and angles change. Thus this option is not true.

Option D; This is not true for the same reason in option C above.

Read more at; https://brainly.com/question/18156103

can someone tell me what to write please

Answers

Answer: 3.5 years

Step-by-step explanation:  Let A be the salaries earned from company A for x years and B for company B for the same years.

A = 31000 + n*3000

B = 38000 + n*1000

To find the case where A = b, set them to each other and solve for n, the number of years.

31000 + n*3000 = 38000 + n*1000

2000n = 7000

n = (7000/2000) = 3.5 years.

A B

0 31000 38000

1 34000 39000

2 37000 40000

3 40000 41000

4 43000 42000

5 46000 43000

6 49000 44000

7 52000 45000

Help I need this pls provide expnation too thx

Answers

There are infinitely many solutions. I took geometry a year ago and it was easy algebra isn’t that hard if you need anymore help let me know
Other Questions
Which statement is a hypothesis that would best address the question "Are rainbow trout eating all the macroinvertebrates in streams and starving out other fish species?" If rainbow trout are eating all the macroinvertebrates, then keeping rainbow trout out of new river habitats will allow native species to recover. If rainbow trout are eating all the macroinvertebrates, then introducing them to other streams will help native species recover. Rainbow trout will eat a variety of macroinvertebrates and native fish species. Rainbow trout hunt and prey upon native fish species. What inspires the seventh man to return to his hometown? 4. The size of a bryophyte is limited because it has no system for transporting water highabove the ground. True or False?a. TRUEb.FALSE What property of addition allow us to change 2+4x into4x + 2 between the first and second lines? According to Bartolom de las Casas, what was an effect of the Spanisharrival in the Americas? You are rolling artwork to ship so it fits perfectly in a cylindrical shipping tube. The volume of the shipping tube is169.56 cubic inches. If the radius of the shipping tube is 1.5 inches, what is the maximum height of the artwork youcan ship and what formula can you use to calculate this height?O 6 inches; height =volume3.14 x (radius)?O 18 inches; height:volume3.14 x radiusO 24 inches, heightvolume3.14 x (radius)236 inches; heightvolume3.14 x radiusO 54 inches; heightvolume3.14 Which of these is caused by bacteria? A) tuberculosis B) influenza C) the common cold D) chicken pox Someone plz help me :( Question 1 of 10Which effect is most likely created by an unreliable narrator?A. A character goes on a difficult quest to achieve an important goal.B. A character changes his mind about something he used to believe.C. A character with a limited understanding may see things others donotD. A character with an interesting personality struggles with adifficult problemSUBMIT I need help on my math I need help on d Please help with this question (-11[(3-25 divided by |-2|+5] show the steps and name the properties you used to solve the equation belowlog3=log2-log(3x+7)Ps. if you just try to get points I will have your answer removed which takes away the points Which of the following is described by the change in an objects position? please helppp1. position2. force3. motion An athlete eats 70 grams of protein per day while training. How much is this in milligrams?Be sure to include the correct unit in your answer. A garden is in the shape of a trapezium whose parallel sides are 40m and 22m and the perpendicular distance between them is 12m. Find the area of the garden Which action is due to field forces Which one of these things did NOT happen on July 4, 1776?A)The Continental Congress made the final changes to the Declaration of Independence.B)The Continental Congress approved the Declaration of Independence.C)The Liberty Bell rang to announce the first public reading of the Declaration of Independence.D)John Hancock signed the Declaration of Independence. Write the expression as a single logarithm.log 2x - log 6y SOmeone help me please i need to finish this